I neeeddddd helppppp!!!! It’s urgentttttt

Answers

Answer 1

Answer:

R theta 1 = 20 x

R theta 2 = 11 x  - 6

R ( theta 1 + theta 2) = 31 x - 6     adding equations

R pi = 31 x - 6       since theta 1 + theta 2 = 180 deg

x = (R pi + 6)  / 31

This equation depends only R

If one lets R be one then   x = (pi + 6) / 31

This would give x = .29489 rad for the value of pi is deg

The numerical value of x appears to depend on the value of R


Related Questions

9/37 is changed to a decimal. What digit lies in the 2005th place to the right of the decimal point?

Answers

Answer:

2

Step-by-step explanation:

Divide 9/37 and you get repeating decimal of 0.243

Divide 2005 by 3 because the decimal repeats 3 numbers

You will get reminder of 1 from dividing 2005 by 3

Move 1 place from the decimal point and you get 2

Thank you so much for your help

Answers

Answer:

1.1x

Step-by-step explanation:

that is the procedure above

Let f(x) = 2x - 7 and g(x) = -6x - 3. Find f(x) + g(x) and state its domain.
HELP PLSSSSS!!!!!!!!!!!!!!!!!!!!!!!!!!!


A : 12x2 - 48x + 21; all real numbers
B: -14x2 + 36x - 18; all real numbers except x = 7
C: 12x2 - 48x + 21; all real numbers except x = 1
D: -14x2 + 36x - 18; all real numbers

Answers

Answer:

Step-by-step explanation:

f(x) + g(x) = 2x -  7 - 6x - 3

f(x) + g(x) = -4x - 10

The domain is any real number.

I think you have mixed up the question. None of the choices are correct. They look like they belong to another choice.

It could be f(x)*g(x)

(2x - 7) (-6x - 3)

-12x^2 - 42x - 6x + 32

-12x^2 - 48x + 21

Well it could be either A or C since they are identical.

Introduction to area of a piecewise rectangular figure

Answers

Given:

The piecewise rectangular figure.

To find:

The area of the piecewise rectangular figure.

Solution:

Draw a line and divide the given figure in two parts (a) and (b) as shown in the below figure.

Figure (a) is a rectangle of length 5 yd and width 3 yd. So, the area of the rectangle is:

[tex]Area=length\times width[/tex]

[tex]A_a=5\times 3[/tex]

[tex]A_a=15[/tex]

Figure (b) is a square of edge 2 yd. So, the area of the square is:

[tex]Area=(edge)^2[/tex]

[tex]A_b=(2)^2[/tex]

[tex]A_b=4[/tex]

The area of the given figure is:

[tex]A=A_a+A_b[/tex]

[tex]A=15+4[/tex]

[tex]A=19[/tex]

Therefore, the area of the given figure is 19 square yd.

Please need help explanation need it

Answers

Answer:

308 m^3

Step-by-step explanation:

The volume is given by

V = l*w*h  where l is the length , w is the width and h is the height

V = 7*4*11

V = 308 m^3

- CA Geometry A Illuminate Credit 4 FF.pdf

Answers

Answer:

hii

Step-by-step explanation:

If a 750 ml bottle of juice costs £1.90 and a 1 litre bottle of the same juice costs £2.50 then the 750 ml bottle is better value.

Answers

Answer:

The 1 liter bottle is better value

Step-by-step explanation:

Cost of 750 ml = £1.90

Cost of 1 liter = £2.50

1000 ml = 1 liter

Cost per 250 ml

750 ml / 3 = £1.90 / 3

250 ml = £0.6333333333333

Approximately,

£ 0.633

Cost per 250 ml

1 liter / 4 = £2.50 / 4

250 ml = £0.625

The 750 ml bottle is not a better value

The 1 liter bottle is better value

What is the period 3 pi and 4 pi

Answers

Answer:

i think i know the answer sorry if im wrong but i would say B

Step-by-step explanation:

Which of the following is a correct tangent ratio for the figure? A) tan (24) 76 B) tan(76°) °= 2 C) tan(76°) = D) tan(8") = 24 76​

Answers

Given question is incorrect; here is the complete question.

"Which of the following is a correct tangent ratio for the figure attached"

A) tan(76°) = [tex]\frac{24}{8}[/tex]

B) tan (76°) = [tex]\frac{8}{24}[/tex]

C) tan (24°) = [tex]\frac{76}{8}[/tex]

D) tan (8°) = [tex]\frac{24}{76}[/tex]

Option A will be the correct option.

   From the figure attached,

Given triangle is a right triangle.Measure of one angle = 76°Measure of two sides of the triangle are 24 and 8units.

By applying tangent ratio of angle having measure 76°.

tan(76°) = [tex]\frac{\text{Opposite side}}{\text{Adjacent side}}[/tex]

             = [tex]\frac{24}{8}[/tex]

    Therefore, Option (A) is the correct option.

Learn more,

https://brainly.com/question/14169279

Please help!

A line intersects the points (-2, 8) and
(4, 12). Find the slope and simplify
completely.
Help Resource
Slope
[?]
= +
Hint: m =
y2-yi
X2-X1
Enter

Answers

Answer:

2/3

Step-by-step explanation:

We can use the slope formula

m = (y2-y1)/(x2-x1)

 = (12-8)/(4 - -2)

  (12-8)/(4+2)

  4/6

   2/3

Determine the period

Answers

Answer:

3 units

Step-by-step explanation:

The period of a wave is the time taken to complete a cycle of motion of the wave

In the given figure, the graduations of the x-axis, which is the usually time axis = 1 unit

At the origin, (0, 0), the vertical displacement of the wave = 0

The maximum value of the wave function is between x = 0 and x = 1

The minimum value of the wave function is between x = 2 and x = 3

At the point (3, 0) the value of the wave function is again 0, and a cycle of the wave is completed

Therefore, the period of the wave = 3 units of the x-variable

convert 4 seconds to hour​

Answers

Answer:

0.00111111 hrs

Step-by-step explanation:

Have a nice day

Answer:

4/3600 = .001111 hr

Step-by-step explanation:

4 seconds * 1 hour            *    1 minute            =    4/3600 = .001111 hr

                    60 minutes          60 seconds

Solve. -7x+1-10x^2=0

Answers

Answer:

[tex]-7x+1-10x^2=0[/tex]

[tex]-10x^2-7x+1=0[/tex]

[tex]quadratic\:equation:-[/tex] [tex]ax^2+bx+c=0[/tex]

[tex]solutions:-\\\\x_{1,\:2}=\frac{-b\pm \sqrt{b^2-4ac}}{2a}[/tex]

[tex]For \\A=-10\\B=-7\\C=1[/tex]

[tex]x_{1,\:2}=\frac{-\left(-7\right)\pm \sqrt{\left(-7\right)^2-4\left(-10\right)\cdot \:1}}{2\left(-10\right)}[/tex]

[tex]\sqrt{\left(-7\right)^2-4\left(-10\right)\cdot \:1}=\sqrt{89}[/tex]

[tex]x_{1,\:2}=\frac{-\left(-7\right)\pm \sqrt{89}}{2\left(-10\right)}[/tex]

[tex]x_1=\frac{-\left(-7\right)+\sqrt{89}}{2\left(-10\right)},\:x_2=\frac{-\left(-7\right)-\sqrt{89}}{2\left(-10\right)}[/tex]

[tex]\frac{-\left(-7\right)+\sqrt{89}}{2\left(-10\right)}=-\frac{7+\sqrt{89}}{20}[/tex]

[tex]\frac{-\left(-7\right)-\sqrt{89}}{2\left(-10\right)}=\frac{\sqrt{89}-7}{20}[/tex]

[tex]x=\frac{\sqrt{89}-7}{20}[/tex]

OAmalOHopeO

On Monday morning at 8:00 a.m. the temperature is – 14 o C. Over the
next 6 hours the temperature rises 6 o C. Between 2:00 p.m. on Monday
and 8:00 a.m. on Tuesday the temperature drops 9 o C. Over the next 6
hours the temperature rises only 4 o C. What is the temperature at 2:00
p.m. on Tuesday?

Answers

Answer : the temperature is -13 degrees Celsius

PLS HELP QUESTION ATTACHED

Answers

Answer:

A

Step-by-step explanation:

the -1 represents the graph going down by 1

juans pencil box measures 6 cm long. if the length of the diagonal is 10 cm what is the width of the pencil box

Answers

Answer:

8 cm

Step-by-step explanation:

We can use the Pythagorean theorem to solve since we have a right triangle

a^2 +b^2 = c^2 where a and b are the legs and c is the hypotenuse

a^2 +6^2 = 10^2

a^2 +36 = 100

a^2 = 100-36

a^2 = 64

Taking the square root of each side

sqrt(a^2) = sqrt(64)

a =8

Answer:

8 cm

Step-by-step explanation:

Use the Pythagorean theorem- [tex]a^{2} +b^{2} =c^{2}[/tex]

leg a: 6cm

leg b: unknown

hypotenuse: 10cm

Therefore [tex]6^{2} +x^{2} =10^{2} = 36+x^{2} =100[/tex]

Subtract 36 to 100 to isolate the [tex]x^{2}[/tex].    [tex]x^{2} =64[/tex]

Square root both sides and get your answer of 8cm

Show Workings.
Question is in attached image.​

Answers

Answer:

A.]A chord of a circle of diameter 40 cm subtends an angle of 70° at the centre of the circle.

Solution given;

diameter [d]=40cm

centre angle [C]=70°

(a) Find the perpendicular distance be tween the chord and the centre of the circle.

Answer:

we have

the perpendicular distance be tween the chord and the centre of the circle=[P]let

we have

P=d Sin (C/2)

=40*sin (70/2)

=22.9cm

the perpendicular distance be tween the chord and the centre of the circle is 22.9cm.

(b) Using = 3.142, find the length of the minor arc.

Solution given;

minor arc=[tex]\frac{70}{360}*πd=\frac{7}{36}*3.142*40[/tex]

=24.44cm

the length of the minor arc. is 24.44cm.

B.]In the diagram, XZ is a diameter of the cir cle XYZW, with centre O and radius 15/2 cm.

If XY = 12 cm, find the area of triangle XYZ.

Solution given:

XY=12cm

XO=15/2cm

XZ=2*15/2=15cm

Now

In right angled triangle XOY [inscribed angle on a diameter is 90°]

By using Pythagoras law

h²=p²+b²

XZ²=XY²+YZ²

15²=12²+YZ²

YZ²=15²-12²

YZ=[tex]\sqrt{81}=9cm[/tex]

:.

base=9cm

perpendicular=12cm

Now

Area of triangle XYZ=½*perpendicular*base

=½*12*9=54cm²

the area of triangle XYZ is 54cm².

Answer:

Question 1

a)

d = 40 cm ⇒ r = 20 cm

Let the perpendicular distance is x.

Connecting the center with  the chord we obtain a right triangle with hypotenuse of r and leg x with adjacent angle of 70/2 = 35°.

From the given we get:

x/20 = cos 35°x = 20 cos 35°x = 16.383 cm (rounded)

b)

The minor arc is 70° and r = 20

The length of the arc is:

s = 2πr*70/360° = 2*3.142*20*7/36 = 24.437 cm (rounded)Question 2

Since XZ is diameter, the opposite angle is the right angle, so the triangle XYZ is a right triangle.

r = 15/2 cm ⇒ XZ = d = 2r = 2*15/2 = 15 cm

Find the missing side, using Pythagorean:

[tex]YZ = \sqrt{XZ^2 - XY^2} = \sqrt{15^2-12^2} = \sqrt{81} = 9[/tex]

The area of the triangle:

A = 1/2*XY*YZ = 1/2*12*9 = 54 cm²

Calculate the answer to the correct number of significant figures: (1.705 + 0.5067) / (0.2 * 1.243) = ______.


8.897


8.8966


8.9


9


8.90

Answers

Answer:

8.9

Step-by-step explanation:

they said to the sig. figure so since it's 8.8966, so the answer will be 8.9

The answer to the correct number of significant figures is 8.897, the correct option is A.

What are Significant Figures?

Significant figures is a positional notation, these are the digits that are required to understand the quantity of something.

The expression is

⇒(1.705 + 0.5067) / (0.2 * 1.243)

=2.2117/0.2486

=8.89662

≈ 8.897

To know more about Significant figures

https://brainly.com/question/14359464

#SPJ2

This drawing would be a step in finding which point of concurrency in a triangle?

Answers

Answer:

B. Orthocenter

Step-by-step explanation:

The orthocenter of a triangle is the point of intersection of the three altitudes (the perpendicular from a vertex to the side facing the vertex) of a triangle

The steps used in constructing the orthocenter includes the steps for drawing of the perpendicular to the sides of the triangle from a point which is the vertex opposite the side of the triangle

Therefore, in the given diagram, a perpendicular to the side KS is drawn from the (vertex) point W, by drawing an arc with center at W that intersects the side KS at two points. Rom the points of intersection of the arc drawn from W intersects KS, with the compass, two arcs are drawn to intersect at a point from which a line drawn to W, is perpendicular to KS

Therefore;

The correct option is B. orthocenter

True or False?
k = 3 over 4 is a solution to the inequality 12k + 2 < 12.
True
False

Answers

Answer:

False.

Step-by-step explanation:

...................

We know that k is 3/4. We can plug in 3/4 into k. 12(3/4) + 2 < 12. This gives us 36/48. 36/48 simplified is 3/4. Our equation is currently 3/4 + 2 < 12. We need common denominators to add 3/4 and 2. 2 with a denominator of 4 is is 8/4. 3/4 + 8/4 is 11/4. 11/4 < 12. 11/4 as a mixed number is 2 3/4. 2 3/4 as a decimal is 2.75. 2.75 < 12. 2.75 is less than 12. This is true.

Evaluate the expression 3(5 + 2)(7 - 2) using order of operations.

Answers

The answer is 36. Do the parentheses first and then multiply by 3

Answer:

105

Step-by-step explanation:

The order of operations is written as PEMDAS. These letters stand for:

-Parentheses

-Exponents

-Multiplication

-Division

-Addition

-Subtraction

We follow these steps in order to solve expressions efficiently. Now, we are going to use PEMDAS to evaluate the expression 3(5+2)(7-2) step by step.

3(7)(5)  The first step is to simplify the numbers in the parentheses.

There are no exponents, so we skip to the next step, multiplication.

(3*7)(5)

21(5)

105

PEMDAS is no longer needed because 105 has come out to be our answer.

I hope this helps you out! Have an an awesome day :3

Help PLEASE eeeeeeeeeeeeeeeee

Answers

Answer:

0.6 is the answer please

which statement must be true about line TU?

Answers

line tu is parallel to rs

Answer:

line TU has no slope in the diagram above

Can I know the answer for the above questions

Answers

Answer:

Step-by-step explanation:

what is 8/9 divide 2/3?

Answers

Answer:

4/3

Step-by-step explanation:

8/9 ÷ 2/3

Simplify the complex fraction.

4/3

Step-by-step explanation:

8/9 ÷ 2/3

Simplify

4/3 is the correct answer

Which statement is true regarding the angles in the figure below?

Triangle E D F. A diagonal line is drawn from point D to form an angle.
Angle D is an exterior angle because it shares a side with the triangle.
Angle D is an exterior angle because it is not inside the triangle.
Angle D is an exterior angle because it is formed by one side of the triangle and by extending another side of the triangle.
Angle D is not an exterior angle.

Answers

An exterior angle is an angle supplementary to one of the interior angles.  

In other words, an exterior angle is the angle between one of the sides, and the extension of an adjacent side.

In the given diagram, the angle D is measured from one of the sides, but not to the extension of an adjacent side.

Therefore angle D is not an exterior angle.

Option D is the correct one!

An exterior angle is an angle supplementary to one of the interior angles.  

We have given that,

Triangle E D F. A diagonal line is drawn from point D to form an angle.

Angle D is an exterior angle because it shares a side with the triangle.

Angle D is an exterior angle because it is not inside the triangle.

Angle D is an exterior angle because it is formed by one side of the triangle and by extending another side of the triangle.

Angle D is not an exterior angle.

What is the exterior angle?

an exterior angle is an angle between one of the sides and the extension of an adjacent side.

In the given diagram, the angle D is measured from one of the sides, but not to the extension of an adjacent side.

Therefore angle D is not an exterior angle.

To learn more about the exterior angle visit:

https://brainly.com/question/24242466

#SPJ5

Please help me with this on the image

Answers

Answer:

hope this may help u

q. no (a)= 60%

(b)=55%

Answer:

a) [tex]\frac{6}{10}[/tex] = [tex]\frac{3}{5}[/tex]  = .6 or 60%

b) [tex]\frac{11}{20}[/tex]  = .55 or 55%

Step-by-step explanation:

Count the boxes.

[tex]\frac{shaded}{total}[/tex]

Divide and multiply by 100

6 – x + = 6 minus StartFraction 3 Over 4 EndFraction x plus StartFraction 1 Over 3 EndFraction equals StartFraction one-half EndFraction x plus 5.X + 5 2 3 6 12

Answers

Answer:

x = 16/15

Step-by-step explanation:

Given:

6 - 3/4x + 1/3 = 1/2x + 5

Collect like terms

6 + 1/3 - 5 = 1/2x + 3/4x

(18+1-15)/3 = (2x+3x)/4

4/3 = 5/4x

x = 4/3 ÷ 5/4

x = 4/3 × 4/5

x = (4 * 4) / (3 * 5)

x = 16/15

(Hurry giving brainliest) Which idea does Arnold need to correct? In specular reflection, reflected rays move in the same direction. In scattering, light strikes particles in the atmosphere. In diffuse reflection, reflected rays move in different directions. In scattering, long wavelengths are scattered the most.

Answers

Answer:

D. In scattering, long wavelengths are scattered the most.

Step-by-step explanation:

Scattering of light is the effect observed when there is an interaction between light energy and some particles. This occurs often in the earth's atmosphere which consists of different sizes of particles that interacts with the sunlight.

In scattering, light of shorter wavelengths are scattered by small sized particles compared to those with longer wavelengths. Example is the scattering of the sunlight by the atmospheric particles where colors with short wavelengths are scattered the most.

Arnold need to correct the statement; In scattering, long wavelengths are scattered the most. To read; In scattering, short wavelengths are scattered the most.

Answer:

D on edgen

Step-by-step explanation:

What is the measure of ∠
A. 6°

B. 42°

C. 60°

D. 49°

Answers

Answer:

<XYZ is equal to 49°

Step-by-step explanation:

Set the two expressions equal to each other so 7x+7=5x+19. Subtract 5x from 7x and 7 from 19 which is equal to 2x=12 that means x is 6. then plug 6 into (7x+7) which is equal to 49.

Other Questions
Select the correct answer. Which graph represents this inequality? y 4x 3 [tex]\sf[/tex] what is meaning of acceleration? write a program to input 3 numbers and print the largest and the smallest number without using if else statement This is the options. Given two integers that represent the miles to drive forward and the miles to drive in reverse as user inputs, create a SimpleCar object that performs the following operations:Drives input number of miles forwardDrives input number of miles in reverseHonks the hornReports car statusThe SimpleCar class is found in the file SimpleCar.java.100 4the output is:beep beepCar has driven: 96 milesimport java.util.Scanner;public class LabProgram { public static void main(String[] args) { Scanner scnr = new Scanner(System.in); /* Type your code here. */ }} 6. If the zeros of a quadratic funtion, f(x), are -2 and 6, what is the equation of the axis ofsymmetry of f(x)?intsa. X= -2b. X= 2c. X= 4d. Cannot be determined https://www.drfrostmaths.com/util-generatekeyskillpic.php?name=AnglePoint4&width=400&params=%5B61%2C0%2C%22y%22%2C4%5D A select list of transactions for Goals follows: For each transaction, identify what type of adjusting entry would be needed. Select from the following four types of adjusting entries: deferred expense, deferred revenue, accrued expense, and accrued revenue. Apr. 1 Paid six months of rent, $4,800. 10 Received $1,200 from customer for six month service contract that began April 1.Apr. 15 Purchased a computer for $1,000. Apr. 18 Purchased $300 of office supplies on account. Apr. 30 Work performed but not yet billed to customer, $500Apr. 30 Employees earned $600 in salaries that will be paid May 2. !!Plz urgent help!!this triangular prism can contain that holds beads that are spherical and each 0.525 cm. How manybeads can fit inside this container?Will mark brainliest The recon quits a was the recon quest of yes or no? please help What does cuchara mean? whats the area of this shape HELP ASAP!!!!!!Thank you so much Why would General Billy Mitchell describe Alaska in the 1930's as the"center" of the world? How could Alaska be both the "center" of theworld and relatively isolated at the same time? An individual in the 36 percent tax bracket has $20,000 invested in a tax-exempt account. If the individual earns 10 percent annually before taxes and inflation is 3.0 percent per year, what is the real value of the investment in 10 years? Which graph shows the solution to the given system of inequalities? [y An Arizona State University student spends his summer months in his hometown. He attends the local community college, where he takes summer courses that he knows will transfer back to ASU. Although he finds the courses just as challenging as at his regular university, he does not worry about his summer grades affecting his 3.2 (out of 4.0) grade point average, because ASU accepts only the transfer credits. As long as he receives C grades or better, he maintains his current grade point average. Which theory suggests that the student will work only as hard as necessary to earn a C grade? In terms of its relations with neighboring powers, the Song Group of answer choices reconquered the northern region controlled by the Uighurs. met their ultimate demise at the hands of the Mongols. were able to maintain a permanent dominance over the Jin Dynasty of the Jurchens. learned from the mistakes of the Tang and avoided a similar fate. conquered Japan, the first time that this had happened. 14. What does the Temporary Assistance for Needy Families program provide?Oretirement income for the elderlycash to workers injured on the jobcompensation to U.S. citizens who lose jobscash to states to help run welfare programs